Übergangs-Marathon Mathematik - Seite 3

Neue Frage »

tmo Auf diesen Beitrag antworten »

Wirklich tolle Aufgabe, finde ich Freude Das ist genau die Art Aufgabe, wie sie hier hingehört. Elementar lösbar, aber trotzdem kein Einzeiler und wahrscheinlich sogar so "selten", dass die meisten (so wie ich z.B.) noch nie drüber nachgedacht haben.

Die Lösung ist:

Ist die eindeutig betimmte natürliche Zahl mit , so wird die Folge schließlich konstant sein, außer (D.h. a fehlt nur eine einzige Zahl zur nächsten Quadratzahl), dann pendelt die Folge schließlich zwischen den beiden Zahlen und .

Beweis:

Zunächst mal folgende Überlegungen:

Es gilt für jede reelle Zahl x: .
Und für jede natürliche Zahl x gilt: .

Dies liefert , also (Wegen der Ganzzahligkeit - man bemerke, dass eine Ungleichung der Kette echt ist). Also ist die Folge ab dem zweiten Folgenglied größergleich N.

Nun schauen wir uns an, was passiert, wenn gilt: Dann gilt , also und somit , also , d.h. die Folge wird echt kleiner.

Zusammen mit obiger Überlegung folgt nun, dass die Folge schließlich den Wert erreichen wird. Die Frage ist nun nur noch, ob sie dann konstant bleibt.

Dort unterscheiden wir nun:

1. Fall: .

Dann gilt , also sind in alle Divisionen schon ganzzahlig, d.h. die Folge pendelt tatsächlich zwischen und .

2. Fall .

Dann gilt , also , also , also , d.h. die Folge bleibt konstant.
Leopold Auf diesen Beitrag antworten »

Es stimmt alles, was du sagst. Wie bei dir nicht anders zu erwarten ...

Ich hatte mir dieses Problem einmal vor längerer Zeit gestellt. Man kann mit dem Ganzzahl-Heron-Verfahren reelle Wurzeln mit vorgegebener Genauigkeit mit Hilfe von Ganzzahlarithmetik bestimmen. Will man etwa die ersten 100 Stellen von berechnen, so setze man in der Folge



Wenn einem nichts Besseres einfällt, startet man bei der Hälfte: . Die Folge fällt zunächst. Wenn zum ersten Mal gilt, ist man am Ziel. Für dieses gilt:



Setzt man nach der führenden ein Komma, hat man danach die ersten 100 korrekten Stellen von .
tmo Auf diesen Beitrag antworten »

Aufgabe 32

Ein Betrunkener verlässt nachts die Kneipe. Er wohnt in der selben (beidseitig unendlich langen) Straße, in der auch die Kneipe steht, und zwar Schritte von der Kneipe entfernt. Er weiß aber leider nicht mehr in welcher Richtung, daher macht er jeden Schritt (unabhängig von seinem vorherigen Schritt) zufällig mit jeweils Wahrscheinlichkeit 1/2 in Richtung seiner Wohnung oder eben von seiner Wohnung weg.

Man berechne die erwartete Anzahl der benötigten Schritte, bis er zu Hause ankommt.
tmo Auf diesen Beitrag antworten »

Ok, ich beende die Aufgabe mal:

sei die Anzahl der nötigen Schritte, offenbar ist .

Wir berechnen zunächst .

Dazu sei die Anzahl der benötigten Schritte bis man entweder bei Schritten in der richtigen Richtung angekommen ist oder bei Schritten in der falschen Richtung angekommen ist. Klar: .

Aber für können wir eine leichte Rekursion finden: mit den Startwarten .

Man sieht leicht, dass diese Rekursion erfüllt, also haben wir insbesondere für alle und somit .

Man beachte, dass die Wahrscheinlichkeit, dass der Betrunkene nach endlicher Zeit zu Hause ankommt, trotzdem 1 ist.


Wer will, wer hat noch nicht?
Mathema Auf diesen Beitrag antworten »

Dann stelle ich hier eine Aufgabe aus dem Bereich der Zahlentheorie:

Zitat:
Aufgabe 33

Zeige:

Für alle natürlichen Zahlen n ist keine Quadratzahl.


Viel Spaß dabei!

@tmo: Dank dir für deine Rückmeldung.
Gast11022013 Auf diesen Beitrag antworten »

Edit:

Naja, jetzt ist es auch egal.

Das wäre mein Vorschlag gewesen:

Zitat:


Lösung: Aufgabe 33

Die Endziffer einer Quadratzahl ist stets 0, 1, 4, 5, 6 oder 9

Man betrachte



Stellt man n als 2m oder 2m+1 dar, so erhält man

1. n=2m



Für m gerade ist



Für m ungerade ist



2. n=2m+1



Für m gerade ist



Für m ungerade ist



Daher hat entweder die Endziffer 3 oder 7 und kann somit keine Quadratzahl sein.
 
 
ollie3 Auf diesen Beitrag antworten »

hallo,
juchu, ich glaube ich habe die lösung gefunden:
Man betrachte die sache modulo 8: 3^n kann nur die werte 1 und 3 annehmen,
2* 17^n nur den wert 2, 3^n+ 2*17^n ist also immer kongruent zu 3 oder 5.
Qudrate modulo 8 können nur die werte 0,1,oder 4 annehmen, also nie 3 oder
5, und das ist glaube ich schon alles Augenzwinkern
gruss ollie3

PS: ich reiche den stab für eine neue aufgabe an jemand anderen weiter..
RavenOnJ Auf diesen Beitrag antworten »


Da hier jetzt schon käftig gelöst wird, 24-Stunden-Regel hin oder her, gebe ich auch noch meinen Senf dazu Augenzwinkern .

Es ist (mit Legendre-Symbolen) . Andererseits . Dies ist inkompatibel für alle natürlichen Zahlen, d.h. ist genau dann quadratischer Rest modulo 3 wenn er quadratischer Nichtrest modulo 17 ist und umgekehrt, der Ausdruck kann also keine Quadratzahl sein.
Nofeykx Auf diesen Beitrag antworten »

Zitat:
Aufgabe 34

Seien, Mengen und bijektiv.
Gib mit Hilfe von eine Bijektion an.
(Verkettungen, Fallunterscheidungen, etc. , alles erlaubt)

Wir arbeiten mit Auswahlaxiom.
tmo Auf diesen Beitrag antworten »

Zunächst mal ein paar notationstechnische Vorbereitungen:

Sei die Projektion.
Ferner versehen wir mit der üblichen additiven Gruppenstruktur (aus rein notationstechnischen Gründen).
Sei die Menge aller Bijektionen von auf sich selbst.

Wir führen hier eine Transpositionsschreibweise ein, d.h. beispielsweise bezeichne diejenige Bijektion aus , die die beiden Elemente und vertauscht und den Rest fixiert.

Das Ziel ist es ein zu finden, derart dass die neue Bijektion folgende Eigenschaft hat:

für alle .

Dass wir dann vermöge eine Bijektion kriegen, ist klar.

Wir konstruieren ein solches mit transfiniter Induktion: Fixiere zunächst eine Wohlordnung auf .

Sei beliebig. Angenommen wir haben ein mit

für alle

gefunden.

Betrachte nun . Wegen obiger Eigenschaft ist .
Dasselbe gilt für das Tupel

Daher gilt mit immer noch

für alle .

Nun gilt allerdings zusätzlich



und somit sind wir mit transfiniter Induktion fertig.

Wenn abzählbar ist, funktioniert das ganze natürlich ohne Auswahlaxiom.
Mathema Auf diesen Beitrag antworten »

Dann stelle ich noch mal eine Aufgabe. Nachdem die letzte Aufgabe wohl eher für den Unimarathon passend war, ist diese vll eher für den Schulmarathon passend. Aber ein wenig nachdenken muss man schon. Viel Spaß dabei.

Zitat:
Aufgabe 35

Auf dem abgebildeten Geobrett (s. Anhang) werden Rechtecke gespannt. Die Seiten der Rechtecke liegen parallel zu den Nagelreihen.

a) Gib die Anzahl unterschiedlicher (d.h. nicht kongruenter) Rechtecke an, die parallel zu den Nagelreihen gespannt werden können. Gib an, wie viele dieser Rechtecke Quadrate sind.

b) Nun soll die Lage der Rechtecke auf dem Geobrett berücksichtigt werden. Gib die Anzahl aller Möglichkeiten an, Rechtecke in unterschiedlicher Position und Größe parallel zu den Nagelreihen aufzuspannen. Gib an, wie viele dieser Rechtecke Quadrate sind.

c) Verallgemeinere die Anzahlbestimmungen auf Geobretter mit n × m Nägeln.

Mathema Auf diesen Beitrag antworten »

Ok - die Aufgabe kam wohl nicht so gut an. Denn will ich nur kurz mal die Lösung skizzieren und gebe danach diesen Thread frei für eine neue Aufgabe.

a) Die Anzahl der nicht kongruenten Rechtecke ist offensichtlich die Summe der Zahlen von 1 bis 6; also nach bekannter Summenformel . Davon sind 6 Quadrate.

b) Fängt man beim Nagel oben links an zu zählen und geht die einzelnen Nägel bzw. Reihen durch, ergibt sich für die Anzahl der Rechtecke folgender Term: . Für die Anzahl der Quadrate gilt: .

c) Sei o.B.d.A n die Höhe und m die Breite, mit n>m.

Analog zu den Überlegungen aus b) ergibt sich für die Möglichkeit aller Rechtecke auf dem Geobrett folgender Term:



Für die Anzahl der Quadrate gilt:



Zur Vereinfachung wird im ersten Faktor m-m addiert:













Ich hoffe ich habe keinen Tippfehler.

Wink

Edit von Guppi12: Zeilenumbruch eingefügt, um Überlänge zu vermeiden
tmo Auf diesen Beitrag antworten »

Aufgabe 36:

Gegeben sei eine quadratische nxn-Matrix, deren Zeilen als n-stellige Zahlen gelesen durch 4 teilbar sind. Man zeige, dass die Determinante dieser Matrix durch 4 teilbar ist.
Guppi12 Auf diesen Beitrag antworten »

Zitat:
Lösung 36


Für ist die Aussage klar.
Für überlegen wir uns folgendes:

Eine zweistellige Zahl ist genau dann durch teilbar, wenn eines der folgenden gilt:

Zehnerstelle gerade, Einerstelle durch 4 teilbar.
Zehnerstelle ungerade, Einerstelle gerade, aber nicht durch 4 teilbar.

Das ist entweder sofort einsichtig oder kann ansonsten in kurzer Zeit überprüft werden.

Sein nun unsere Matrix. Es sind also und durch teilbar. Ist gerade, so ist durch teilbar, somit auch . Da gerade ist und gerade ist, ist auch durch teilbar. Dann ist auch die Determinante obiger Matrix, nämlich durch teilbar.

Selbiges gilt natürlich auch, wenn gerade ist. Es bleibt also der Fall, dass beide ungerade sind. Dann lassen aber nach obiger Charakterisierung jeweils Rest bei Division durch , was dann auch für zutrifft. Dann lässt aber Rest .

Damit ist der Beweis für abgeschlossen.

Ist entwickeln wir nach der ersten Spalte der Matrix. Dabei überträgt sich die Teilbarkeit durch aller Zeilen der Gesamtmatrix auf die kleineren Matrizen, bei denen je eine Zeile und Spalte gestrichen wurde (gibt es für diese eigentlich eine Bezeichnung ?),
weil die Teilbarkeit durch einer Zahl nur von den letzten beiden Ziffern abhängt und diese nicht modifiziert werden. Induktiv folgt die Behauptung, da die Determinante der großen Matrix eine -Linearkombination der Determinanten der kleineren Matrizen ist.


Ein schönes Beispiel dafür, dass bei Induktionsbeweisen nicht immer der Induktionsschritt das Aufwändige ist, sondern es auch mal der Anfang sein kann.

Werde, wenn es keine Einwände gibt, gleich eine neue Aufgabe posten.
Guppi12 Auf diesen Beitrag antworten »

Zitat:
Aufgabe 37
Betrachte folgendes Objekt:

[attach]35634[/attach]

Jeder Buchstabe markiert einen Schnittpunkt der 4 gezeichneten Kreise (die Buchstaben dienen nur der Identifizierung, damit man im Beweis leichter darüber sprechen kann). Auf diese Schnittpunkte sollen jetzt die Zahlen 2-10 derart verteilt werden, dass die Summe der Zahlen auf dem blauen, roten, grünen, gelben Kreis jeweils gleich sind. Wieviele Möglichkeiten, bis auf Spiegelungen und Drehungen, dies zu tun gibt es ?

Nofeykx Auf diesen Beitrag antworten »

Zitat:
Lösung 37

Antwort: Es gibt bis auf Drehung und Spiegelung genau 16 verschiedene passende Konfigurationen.

Beweis:

Zunächst noch einmal das Bild:

[attach]35634[/attach]

Sei die Summe der Zahlen, die am Ende auf jedem Kreis liegen soll.
Zunächst stellen wir fest, dass jede Zahl auf genau Kreisen liegt.
Daraus folgt, indem wir alle Kreise aufaddieren (und somit jede Zahl genau doppelt zählen)

, bzw. .

Auf dem blauen Kreis liegen nur Zahlen. Da die größtmögliche Summe aus Zahlen genau ist, müssen auf dem blauen Kreis liegen. Die Anordnung der Zahlen spielt dabei keine Rolle, da sich so bloß Spiegelungen und Drehungen ergeben. Legen wir hingegen eine Reihenfolge fest, so ist jede Möglichkeit, die wir mit dieser Festlegung herausfinden nicht Spiegelung oder Drehung einer anderen. Wir legen also

,
,
fest.

Desweiteren stellen wir fest, dass wir, wann immer wir die Felder , bzw. , bzw. belegen müssen, durch Vertauschung zwei völlig gleichwertige Möglichkeiten dafür haben. Wir werden uns in diesem Fall deswegen immer für eine Möglichkeit entscheiden und zum Schluss die Gesamtzahl der möglichen Konfigurationen mit multiplizieren.

Soviel zur Vorüberlegung. Jetzt ans Eingemachte:

Wir stellen fest, dass auf dem roten Kreis die liegen muss, denn würde sie es nicht, so wäre die größtmögliche Summe auf dem roten Kreis , also zu klein.

Weiter unterscheiden wir nun zwei Fälle:

1. Fall: Die 7 liegt nicht auf dem grünen Kreis:
In diesem Fall ist die größtmögliche Summe auf dem grünen Kreis genau .

Das heißt aber auch, dass genau diese größtmöglichen Ziffern auf dem grünen Kreis liegen müssen. Dann liegen nur die Ziffern und nicht auf dem grünen Kreis. Diese müssen dann auf liegen. Wir setzen . Damit liegt auf dem roten Kreis bisher eine Summe von . Es fehlen noch . Das ist nur möglich, wenn sich der rote Kreis die und die mit dem grünen Kreis teilt. Also müssen auf die und die liegen. Wir setzen . Es bleiben noch , die wir auf verteilen durch .

Wir haben also und rechnen nach, dass diese Konfiguration tatsächlich richtig ist.


2. Fall: Die liegt auf dem grünen Kreis:
Da die auf dem grünen und roten Kreis liegt, kann sie nicht auf dem gelben Kreis liegen.
Für den gelben Kreis bleiben dann bloß noch die Zahlen übrig. Von diesen müssen wir
auswählen, deren Summe ist. Man sieht sehr leicht, dass das nur mit möglich ist.

Auf dem gelben Kreis liegen also die Zahlen .
Das heißt, die einzigen verbleibenden Zahlen, nämlich die und die müssen auf liegen. Wir setzen . Damit liegt nun auf dem roten Kreis eine Summe von . Auf dem roten Kreis, bleiben damit noch zu belegen, die in Summe sein müssen. Wir müssen also zwei Zahlen aus (den Zahlen auf dem gelben Kreis) auswählen, deren Summe ist. Da wir eine ungerade Zahl brauchen, ist dabei. Damit muss die andere Zahl sein. Wir legen also fest . Damit bleibt nur noch eine Wahl, nämlich .

Wir haben also und rechnen nach, dass auch diese Konfiguration tatsächlich richtig ist.

Wir haben also 2 verschiedene mögliche Konfigurationen gefunden. Insgesamt gibt es also mögliche verschiedene Konfigurationen, die alle nicht durch Drehung oder Spiegelung ineinander überführbar sind.

tmo Auf diesen Beitrag antworten »

Ich habe gerade etwas parat, daher nehme ich das Angebot mal an:

Aufgabe 38

Es sei ein quadratisches Polynom mit ganzzahligen Koeffizienten. Angenommen sei Quadratzahl für alle .

Man zeige, dass dann selbst das Quadrat eines linearen Polynoms mit ganzzahligen Koeffizienten ist.
HAL 9000 Auf diesen Beitrag antworten »

Na das geht ja ganz konventionell die Tippel-Tappel-Tour:

Zitat:
Lösung zu Aufgabe 38

Wenn alle Quadratzahlen sein sollen, dann muss dies auch auf alle



zutreffen.


1) Im Fall kriegen wir



für genügend große (konkret nach Umstellung: für alle ). Diese können unmöglich Quadratzahlen sein, da sie zwischen zwei benachbarten Quadratzahlen liegen.


2) Im Fall leistet das ganze



für alle .


Bleibt nur übrig, insbesondere ist dann gerade mit und folglich .

Werde jetzt keine neue Aufgabe finden, also wer will...
tmo Auf diesen Beitrag antworten »

Schöne Lösung, die sich von meiner sogar deutlich unterscheidet. (Ist ja deutlich interessanter eine noch nicht bekannte Lösung zu sehen smile )

Wie wärs hiermit?

Aufgabe 39

Sei . Man finde alle Funktionen , die der Funktionalgleichung genügen und darüber hinaus der Quotient auf beschränkt ist.
HAL 9000 Auf diesen Beitrag antworten »

Zitat:
Lösung zu Aufgabe 39

Mit ein bisschen Probiererei sieht man schnell, dass eine mögliche Lösungsfunktion ist. Kommt der schwierigere Teil nachzuweisen, dass es keine weitere gibt:

Betrachten wir , dann wird obige Rekursionsgleichung zu




1) Angenommen, es existiert ein mit . Dann folgt leicht per Induktion

für alle .

Dann haben wir aber im Widerspruch zur genannten Beschränktheitsvorraussetzung für .


2) Angenommen, es existiert ein mit . Unter Zuhilfenahme von folgt dann per Induktion

für alle ,

dabei nutzt man im Induktionsschritt .

Für genügend große widerspricht das aber , da der quadratische Term in (*) schneller fällt als jeder lineare Term.
tmo Auf diesen Beitrag antworten »

Da ja niemand will, hier die Aufgabe 40:

Man finde alle Tripel natürlicher Zahlen mit





Jetzt dürfen sich aber auch mal alle außer HAL anstrengen Augenzwinkern
Mathema Auf diesen Beitrag antworten »

Dann hier mal ein Lösungsversuch, bevor HAL seine "Drohung" wahrmacht.

Zitat:
Lösung zu Aufgabe 40:

Seien a, b, c natürliche paarweise tellerfremde Zahlen mit
mod a
mod b
mod c

Dann gilt:

mod a
mod b
mod c

Also auch:



Ferner gilt somit:

mod abc

Daraus folgt (dank an tmo [siehe Diskussionsthread]):

(mod abc)

Somit muss größer gleich 1 sein. Das einzige Lösungstripel, welches diese Voraussetzungen erfüllt, ist {2; 3; 5}.
Mathema Auf diesen Beitrag antworten »

Zitat:
Aufgabe 41:

Gegeben ist die Funktion f mit .

Bestimme die Ableitungsfunktion mit Hilfe der h-Methode.

Guppi12 Auf diesen Beitrag antworten »

Zitat:
Lösung 41
Sei und mit . Es gilt

.

Jetzt nutzen wir . Daraus folgt



.

Es folgt .
Guppi12 Auf diesen Beitrag antworten »

Zitat:
Aufgabe 42

Zeige, dass eine Nullstelle des Polynoms ist.
Dabei sollen keine exakten Werte von Cosinus oder Sinus vorausgesetzt werden, außer deren - bzw. -Stellen.


Es gibt mindestens einen (meiner Meinung nach) eleganten Beweis, ohne viel Rechnerei.
Mathema Auf diesen Beitrag antworten »

Zitat:
Lösung zu Aufgabe 42:







Es gilt:

sowie

Sei :









, mit den Lösungen:



Somit gilt:





Nach dem Satz von Vieta gilt:





Somit ist Nullstelle des Polynoms .

Mathema Auf diesen Beitrag antworten »

Zitat:
Aufgabe 43:

Es seien x und y natürliche Zahlen.
Zeige:
Für alle ungeraden natürlichen Zahlen n gilt: .

Guppi12 Auf diesen Beitrag antworten »

Zitat:

Lösung 43

Ist ungerade, so gilt .

Daher folgt



.

Also ist durch teilbar.

Guppi12 Auf diesen Beitrag antworten »

Ist jetzt nichts exotisches. So schöne Aufgaben wie die mit dem unbegrenzten Schachbrett von tmo habe ich leider nicht Lesen2

Zitat:

Aufgabe 44

Sei ein Körper. Zeige:

Ist eine endliche Untergruppe von , so gilt .

tmo Auf diesen Beitrag antworten »

Lösung 44

Ist , so gilt für alle . Insbesondere folgt die Behauptung, da wir wählen können.
tmo Auf diesen Beitrag antworten »

Aufgabe 45

Man zeige, dass die einzige Untergruppe vom Index 2 in ist.

Bemerkung: Dieser Satz gibt einem eine natürliche Definition des Vorzeichens einer Permutation. Denn die übliche Definition greift ja auf eine Ordnung auf der Menge zurück. Das ist ja einfach nur willkürlich und unschön.
Mathema Auf diesen Beitrag antworten »

Zitat:
Lösung Aufgabe 45:

Sei eine Untergruppe mit , dann ist Normalteiler. Die Abbildung von auf die multiplikative Gruppe ist ein surjektiver Homomorphismus.
Da Normalteiler ist, enthält entweder alle Transpositionen, oder gar keine Transposition. Da aber alle Transpositionen erzeugen und eine Untergruppe ist, muss keine Transposition enthalten.
Aufgrund der Homomorphismus-Eigenschaft liegen also alle Produkte von einer geraden Anzahl von Transpositionen in , und das sind die Elemente der Gruppe . Somit ist die einzige Untergruppe von mit Index 2.
RavenOnJ Auf diesen Beitrag antworten »

tmo hat ja eigentlich die Lösung schon im wesentlichen geschrieben, ich vervollständige nur und fasse zusammen.

Lösung 45

wird von allen Paaren von Transpositionen erzeugt. Eine andere Erzeugermenge sind die Dreizyklen: mit paarweise ungleichen . Die Gleichheiten und (Reihenfolge rechts nach links) zeigen dies. Alle Dreizyklen haben Ordnung 3.

Man benötigt nun folgenden Hilfssatz: Sei eine Gruppe und Normalteiler von mit Index n. Dann enthält alle Elemente aus , deren Ordnung teilerfremd zu n ist.
Beweis: Sei ein solches Element mit teilerfremd zu , das nicht in enthalten ist. Weil das Element der Faktorgruppe eine Ordnung haben muss, die Teiler von ist und gleichzeitig Teiler von (wegen Lagrange), ergäbe sich ein Widerspruch.

Sei nun Untergruppe vom Index 2. Da alle Dreizyklen Ordnung 3 haben, teilerfremd zu 2, sind die Dreizyklen Elemente von , also . Da aber schon Index 2 hat, muss sein. ist also die einzige Untergruppe vom Index 2 in .
Mathema Auf diesen Beitrag antworten »

Also wenn keiner eine Aufgabe stellen möchte, dann stelle ich mal eine. Oberstufenmathematik - es sind also auch alle Schüler herzlich eingeladen. Augenzwinkern

Zitat:
Aufgabe 46:

Von einem Kegel sind Radius und Höhe (und damit auch der Schenkel ) gegeben. Ihm soll ein Zylinder (Grundkreis des Zylinders im Grundkreis des Kegels; Variablen: , ) mit aufgesetzter Kugel (Variable: ) einbeschrieben werden, so daß das Gesamtvolumen (Zylinder + Kugel) möglichst groß wird.
Berechne , und .
Mathema Auf diesen Beitrag antworten »

Zitat:
Lösung Aufgabe 46:

Es gilt:



2. Strahlensatz:









Ähnliche Dreiecke:













Es gilt also:





Dann leiten wir mal ab:





Notwendige Bedingung für die Extremstelle:





, liefert also ein Minimum









Nun überprüfen wir noch die hinreichende Bedingung:





, also haben wir ein Maximum gefunden.

Dann bestimmen wir noch und :





Nicht gefordert, aber für das maximale Volumen ergibt sich also:






Hoffentlich ohne Tippfehler. Wer will rechnet mal nach.

Edit Equester: Nutze ', statt ´ (also das Zeichen neben den Enter), dann gibt es keine Konflikte mit Latex. Korrigiert.
Guppi12 Auf diesen Beitrag antworten »

Ich nehme mal das Angebot an und stelle eine neue Aufgabe.

Zitat:
Aufgabe 47

Gegeben sei ein 15-er Puzzle:

[attach]36522[/attach]

Regeln sind denke ich klar, man kann Zahlen nur dann verschieben, wenn sie neben der Lücke liegen.

Es gilt als gelöst, wenn es so aussieht, wie auf dem Bild.

Zeige, dass es nicht lösbar ist, wenn man die 1 mit der 2 vertauscht.
Mathema Auf diesen Beitrag antworten »

Zitat:
Lösung Aufgabe 47:

Es ist leicht zu erkennen, dass das freie Feld bei Start- und Zielstellung an der gleichen Stelle ist. Somit ist die Lösung nur nach einer geraden Anzahl von Zügen möglich. Fassen wir die Startstellung (und jede andere natürlich auch) als ein Element der symmetrischen Gruppe auf. Dann ist die Startstellung folgende Permutation:



Die Zielstellung ist die Identität. Die Startstellung hat eine Fehlstellung, es gilt somit:



Ferner gilt:



Ein Zug im Spiel entspricht einer Transposition mit dem freien Feld (Nummer: 16). Man müsste also mit einer geraden Anzahl von Transpositionen verknüpfen, um zur Identität zu gelangen. Das ist aber unmöglich.
Mathema Auf diesen Beitrag antworten »

Dann noch eine alte Klausuraufgabe aus dem 12. Jahrgang:

Zitat:
Aufgabe 48 (Analysis):

Berechne die Länge des Graphens von über dem Intervall .
bijektion Auf diesen Beitrag antworten »

Zitat:

Lösung Aufgabe 48

Sei . Offenbar ist differenzierbar auf mit .

Für die Länge des Graphens gilt und wegen folgt

.

Mit und ergibt sich dann

.

bijektion Auf diesen Beitrag antworten »

Zitat:

Aufgabe 49

Man bestimme eine Zahl , sodass gilt.
Neue Frage »
Antworten »



Verwandte Themen

Die Beliebtesten »
Die Größten »
Die Neuesten »